SlideShare a Scribd company logo
1 of 74
PROJECTILE MOTION
   constant acceleration in a straight line; free fall under gravity, projectile motion; Relative motion, change in velocity, velocity vector
   components, circular motion (constant speed with one force only providing centripetal force).

 1. Recall that in the absence of friction a falling object will have a constant
    acceleration of 10 ms-2 and that this value is referred to as “gravitational
    acceleration, g”
 2. Use F = mg to show that Nkg-1 is an equivalent unit to ms-2.
 3. Explain the effect that air friction has on the acceleration of a falling object
    (resulting in free fall)

 4. Define the term projectile.
 5. Describe projectile motion
    in terms of its uniform
    horizontal motion and it
    accelerated vertical motion.
 6. Use equations of motion to
    calculate time, distance,
    velocity and acceleration.




Monday, 24 May 2010
FREE FALL IN THE ABSENCE OF FRICTION

Mass & weight
The mass of an object, m is a measure of the amount of matter in that object.

The weight of an object, Fw is a measure of the force due to gravity on that object.

Gravitational constant, g
The force exerted on an object by the earth’s gravitational field is called the
gravitational constant, g.
                                                    What
g = 10 Nkg       -1
                                                   does this
                                                    mean?
This means that for every kg of mass there is a force of 10N acting on it because of
gravity

This causes all objects to fall with an acceleration of 10 ms-2

We can also say g = 10 ms-2 which is called gravitational acceleration




                               Fw = mg
Monday, 24 May 2010
FOR AN OBJECT THROWN VERTICALLY UPWARDS THE ACCELERATION IS CONSTANT
& NEGATIVE
                                                It slows down as it moves in the upwards
 +     up
                                                (positive) direction
                                                and
 -     down
                                                speeds up as it moves in the downwards
                                                (negative) direction.




                      Velocity - time graph for the object

v (ms-1)




                                                             t (s)
                                                                       Constant negative
                                                                       slope shows a
                                                                       constant negative
                                                                       acceleration
Monday, 24 May 2010
FOR AN OBJECT THROWN VERTICALLY UPWARDS THE ACCELERATION IS CONSTANT
& NEGATIVE
                                                It slows down as it moves in the upwards
 +     up
                                                (positive) direction
                                                and
 -     down
                                                speeds up as it moves in the downwards
                                                (negative) direction.




                      Velocity - time graph for the object

v (ms-1)




                                                             t (s)
                                                                       Constant negative
                                                                       slope shows a
                                                                       constant negative
                                                                       acceleration
Monday, 24 May 2010
FOR AN OBJECT THROWN VERTICALLY UPWARDS THE ACCELERATION IS CONSTANT
& NEGATIVE
                                                It slows down as it moves in the upwards
 +     up
                                                (positive) direction
                                                and
 -     down
                                                speeds up as it moves in the downwards
                                                (negative) direction.




                      Velocity - time graph for the object

v (ms-1)




                                                             t (s)
                                                                       Constant negative
                                                                       slope shows a
                                                                       constant negative
                                                                       acceleration
Monday, 24 May 2010
FOR AN OBJECT THROWN VERTICALLY UPWARDS THE ACCELERATION IS CONSTANT
& NEGATIVE
                                                It slows down as it moves in the upwards
 +     up
                                                (positive) direction
                                                and
 -     down
                                                speeds up as it moves in the downwards
                                                (negative) direction.



                                   A symmetrical path


                      Velocity - time graph for the object

v (ms-1)




                                                             t (s)
                                                                       Constant negative
                                                                       slope shows a
                                                                       constant negative
                                                                       acceleration
Monday, 24 May 2010
FOR AN OBJECT THROWN VERTICALLY UPWARDS THE ACCELERATION IS CONSTANT
& NEGATIVE
                                                It slows down as it moves in the upwards
 +     up
                                                (positive) direction
                                                and
 -     down
                                                speeds up as it moves in the downwards
                                                (negative) direction.



                                   A symmetrical path


                      Velocity - time graph for the object

v (ms-1) x




                                                             t (s)
                                                                       Constant negative
                                                                       slope shows a
                                                                       constant negative
                                                                       acceleration
Monday, 24 May 2010
FOR AN OBJECT THROWN VERTICALLY UPWARDS THE ACCELERATION IS CONSTANT
& NEGATIVE
                                                It slows down as it moves in the upwards
 +     up
                                                (positive) direction
                                                and
 -     down
                                                speeds up as it moves in the downwards
                                                (negative) direction.



                                   A symmetrical path


                      Velocity - time graph for the object

v (ms-1) x



                                    x
                                                             t (s)
                                                                       Constant negative
                                                                       slope shows a
                                                                       constant negative
                                                                       acceleration
Monday, 24 May 2010
FOR AN OBJECT THROWN VERTICALLY UPWARDS THE ACCELERATION IS CONSTANT
& NEGATIVE
                                                It slows down as it moves in the upwards
 +     up
                                                (positive) direction
                                                and
 -     down
                                                speeds up as it moves in the downwards
                                                (negative) direction.



                                   A symmetrical path


                      Velocity - time graph for the object

v (ms-1) x



                                    x
                                                             t (s)
                                                                       Constant negative
                                                      x                slope shows a
                                                                       constant negative
                                                                       acceleration
Monday, 24 May 2010
FOR AN OBJECT THROWN VERTICALLY UPWARDS THE ACCELERATION IS CONSTANT
& NEGATIVE
                                                It slows down as it moves in the upwards
 +     up
                                                (positive) direction
                                                and
 -     down
                                                speeds up as it moves in the downwards
                                                (negative) direction.



                                   A symmetrical path


                      Velocity - time graph for the object

v (ms-1) x



                                    x
                                                             t (s)
                                                                       Constant negative
                                                      x                slope shows a
                                                                       constant negative
                                                                       acceleration
Monday, 24 May 2010
FOR AN OBJECT THROWN VERTICALLY UPWARDS THE ACCELERATION IS CONSTANT
& NEGATIVE
                                                 It slows down as it moves in the upwards
 +     up
                                                 (positive) direction
                                                 and
 -     down
                                                 speeds up as it moves in the downwards
                                                 (negative) direction.



                                    A symmetrical path


                      Velocity - time graph for the object

                          Decreasing speed in
v (ms-1) x
                          a positive direction



                                    x
                                                             t (s)
                                                                        Constant negative
                                                       x                slope shows a
                                                                        constant negative
                                                                        acceleration
Monday, 24 May 2010
FOR AN OBJECT THROWN VERTICALLY UPWARDS THE ACCELERATION IS CONSTANT
& NEGATIVE
                                                  It slows down as it moves in the upwards
 +     up
                                                  (positive) direction
                                                  and
 -     down
                                                  speeds up as it moves in the downwards
                                                  (negative) direction.



                                    A symmetrical path


                      Velocity - time graph for the object

                          Decreasing speed in
v (ms-1) x
                          a positive direction

                                                 Stationary for an instant
                                    x
                                                                t (s)
                                                                             Constant negative
                                                          x                  slope shows a
                                                                             constant negative
                                                                             acceleration
Monday, 24 May 2010
FOR AN OBJECT THROWN VERTICALLY UPWARDS THE ACCELERATION IS CONSTANT
& NEGATIVE
                                                    It slows down as it moves in the upwards
 +     up
                                                    (positive) direction
                                                    and
 -     down
                                                    speeds up as it moves in the downwards
                                                    (negative) direction.



                                    A symmetrical path


                      Velocity - time graph for the object

                          Decreasing speed in
v (ms-1) x
                          a positive direction

                                                 Stationary for an instant
                                    x
                                                                t (s)
                                                                             Constant negative
                            Increasing speed in a                            slope shows a
                            negative direction            x
                                                                             constant negative
                                                                             acceleration
Monday, 24 May 2010
Examples
1. A plane drops a Red Cross package from a height of 1200 m. If the package had no
   parachute (and by this you can assume negligible air friction & g = 10 ms-2)
  (a) How fast will the package be travelling just before it hits the ground?
  (b) How many seconds will the package take to fall?




2. A 1 kg object is dropped from a tower 120 m high.
  (a) Calculate the time it will take for the object to fall to the ground.
  (b) Calculate the objects final speed on reaching the ground.
  (c) How long does it take to reach a speed of 35 ms-1 ?




Monday, 24 May 2010
3. A shell is fired straight up with an initial speed of 96 ms-1.
    (a) Calculate the time it will take for the object to fall to the ground.
    (b) When will the shell have an upwards speed of 48 ms-1 ?
    (c) Calculate the time for the shell to reach its maximum height.
    (d) Calculate the maximum height reached by the shell.
    (e) What is the shells acceleration at the top of its motion?




Monday, 24 May 2010
FREE FALL - IN THE PRESENCE OF FRICTION

If the object is falling fast:
Air friction is too large to ignore. Here we consider the object to have two forces
acting on it. Air friction and gravity.
A. When the object is initially falling slowly, air friction is much smaller than the force
   of gravity on the object.
B. As the object speeds up the air friction increases.
C. The object will eventually reach a speed at which the air friction balances the force
   of gravity on the object. The object cannot fall any faster than at this speed (unless
   it changes shape). This speed is called terminal velocity.
This situation is illustrated by the following example:

                                    B
                         A




                                     C



                                                              Vector arithmetic with notes
Monday, 24 May 2010
WHAT IS A PROJECTILE?


Brainstorm


                                      Examples
                                      of
                                      Projectiles




  “What do all these examples have in common?”




                      “Why do you think that a ball dropping vertically (in free fall) is not
                      an example of projectile motion?”




Monday, 24 May 2010
PROJECTILES - HORIZONTAL AND VERTICAL MOTION
• A projectile is an object that has constant horizontal velocity and constant vertical
  acceleration due to gravity. In other words it is moving horizontally at the same time it is in
  free fall. The projectiles velocity is the sum of these two velocities.
• The path of a projectile is parabolic and symmetrical
• Air resistance is considered to be negligible
• The force of gravity is the only force acting on the object. Once the projectile is launched there
  is no thrust force.




                                                  v = the size of the velocity of the projectile
  To determine the velocity:   vx                 vx = the horizontal component of the
                                    vy                 projectile’s velocity
  add the horizontal and
                               v                  vy = the vertical component of the
  vertical components
                                                       projectile's velocity
Monday, 24 May 2010
PROJECTILES - HORIZONTAL AND VERTICAL MOTION
• A projectile is an object that has constant horizontal velocity and constant vertical
  acceleration due to gravity. In other words it is moving horizontally at the same time it is in
  free fall. The projectiles velocity is the sum of these two velocities.
• The path of a projectile is parabolic and symmetrical
• Air resistance is considered to be negligible
• The force of gravity is the only force acting on the object. Once the projectile is launched there
  is no thrust force.




                                                  v = the size of the velocity of the projectile
  To determine the velocity:   vx                 vx = the horizontal component of the
                                    vy                 projectile’s velocity
  add the horizontal and
                               v                  vy = the vertical component of the
  vertical components
                                                       projectile's velocity
Monday, 24 May 2010
PROJECTILES - HORIZONTAL AND VERTICAL MOTION
• A projectile is an object that has constant horizontal velocity and constant vertical
  acceleration due to gravity. In other words it is moving horizontally at the same time it is in
  free fall. The projectiles velocity is the sum of these two velocities.
• The path of a projectile is parabolic and symmetrical
• Air resistance is considered to be negligible
• The force of gravity is the only force acting on the object. Once the projectile is launched there
  is no thrust force.




                                                  v = the size of the velocity of the projectile
  To determine the velocity:   vx                 vx = the horizontal component of the
                                    vy                 projectile’s velocity
  add the horizontal and
                               v                  vy = the vertical component of the
  vertical components
                                                       projectile's velocity
Monday, 24 May 2010
PROJECTILES - HORIZONTAL AND VERTICAL MOTION
• A projectile is an object that has constant horizontal velocity and constant vertical
  acceleration due to gravity. In other words it is moving horizontally at the same time it is in
  free fall. The projectiles velocity is the sum of these two velocities.
• The path of a projectile is parabolic and symmetrical
• Air resistance is considered to be negligible
• The force of gravity is the only force acting on the object. Once the projectile is launched there
  is no thrust force.




                                                  v = the size of the velocity of the projectile
  To determine the velocity:   vx                 vx = the horizontal component of the
                                    vy                 projectile’s velocity
  add the horizontal and
                               v                  vy = the vertical component of the
  vertical components
                                                       projectile's velocity
Monday, 24 May 2010
PROJECTILES - HORIZONTAL AND VERTICAL MOTION
• A projectile is an object that has constant horizontal velocity and constant vertical
  acceleration due to gravity. In other words it is moving horizontally at the same time it is in
  free fall. The projectiles velocity is the sum of these two velocities.
• The path of a projectile is parabolic and symmetrical
• Air resistance is considered to be negligible
• The force of gravity is the only force acting on the object. Once the projectile is launched there
  is no thrust force.




                                                  v = the size of the velocity of the projectile
  To determine the velocity:   vx                 vx = the horizontal component of the
                                    vy                 projectile’s velocity
  add the horizontal and
                               v                  vy = the vertical component of the
  vertical components
                                                       projectile's velocity
Monday, 24 May 2010
PROJECTILES - HORIZONTAL AND VERTICAL MOTION
• A projectile is an object that has constant horizontal velocity and constant vertical
  acceleration due to gravity. In other words it is moving horizontally at the same time it is in
  free fall. The projectiles velocity is the sum of these two velocities.
• The path of a projectile is parabolic and symmetrical
• Air resistance is considered to be negligible
• The force of gravity is the only force acting on the object. Once the projectile is launched there
  is no thrust force.




                                                  v = the size of the velocity of the projectile
  To determine the velocity:   vx                 vx = the horizontal component of the
                                    vy                 projectile’s velocity
  add the horizontal and
                               v                  vy = the vertical component of the
  vertical components
                                                       projectile's velocity
Monday, 24 May 2010
PROJECTILES - HORIZONTAL AND VERTICAL MOTION
• A projectile is an object that has constant horizontal velocity and constant vertical
  acceleration due to gravity. In other words it is moving horizontally at the same time it is in
  free fall. The projectiles velocity is the sum of these two velocities.
• The path of a projectile is parabolic and symmetrical
• Air resistance is considered to be negligible
• The force of gravity is the only force acting on the object. Once the projectile is launched there
  is no thrust force.




                           vx




                                                  v = the size of the velocity of the projectile
  To determine the velocity:    vx                vx = the horizontal component of the
                                     vy                projectile’s velocity
  add the horizontal and
                                v                 vy = the vertical component of the
  vertical components
                                                       projectile's velocity
Monday, 24 May 2010
PROJECTILES - HORIZONTAL AND VERTICAL MOTION
• A projectile is an object that has constant horizontal velocity and constant vertical
  acceleration due to gravity. In other words it is moving horizontally at the same time it is in
  free fall. The projectiles velocity is the sum of these two velocities.
• The path of a projectile is parabolic and symmetrical
• Air resistance is considered to be negligible
• The force of gravity is the only force acting on the object. Once the projectile is launched there
  is no thrust force.




                                          vx




                           vx




                                                  v = the size of the velocity of the projectile
  To determine the velocity:    vx                vx = the horizontal component of the
                                     vy                projectile’s velocity
  add the horizontal and
                                v                 vy = the vertical component of the
  vertical components
                                                       projectile's velocity
Monday, 24 May 2010
PROJECTILES - HORIZONTAL AND VERTICAL MOTION
• A projectile is an object that has constant horizontal velocity and constant vertical
  acceleration due to gravity. In other words it is moving horizontally at the same time it is in
  free fall. The projectiles velocity is the sum of these two velocities.
• The path of a projectile is parabolic and symmetrical
• Air resistance is considered to be negligible
• The force of gravity is the only force acting on the object. Once the projectile is launched there
  is no thrust force.


                                                       vx

                                          vx




                           vx




                                                  v = the size of the velocity of the projectile
  To determine the velocity:    vx                vx = the horizontal component of the
                                     vy                projectile’s velocity
  add the horizontal and
                                v                 vy = the vertical component of the
  vertical components
                                                       projectile's velocity
Monday, 24 May 2010
PROJECTILES - HORIZONTAL AND VERTICAL MOTION
• A projectile is an object that has constant horizontal velocity and constant vertical
  acceleration due to gravity. In other words it is moving horizontally at the same time it is in
  free fall. The projectiles velocity is the sum of these two velocities.
• The path of a projectile is parabolic and symmetrical
• Air resistance is considered to be negligible
• The force of gravity is the only force acting on the object. Once the projectile is launched there
  is no thrust force.


                                                       vx

                                          vx
                                                                              vx



                           vx




                                                  v = the size of the velocity of the projectile
  To determine the velocity:    vx                vx = the horizontal component of the
                                     vy                projectile’s velocity
  add the horizontal and
                                v                 vy = the vertical component of the
  vertical components
                                                       projectile's velocity
Monday, 24 May 2010
PROJECTILES - HORIZONTAL AND VERTICAL MOTION
• A projectile is an object that has constant horizontal velocity and constant vertical
  acceleration due to gravity. In other words it is moving horizontally at the same time it is in
  free fall. The projectiles velocity is the sum of these two velocities.
• The path of a projectile is parabolic and symmetrical
• Air resistance is considered to be negligible
• The force of gravity is the only force acting on the object. Once the projectile is launched there
  is no thrust force.


                                                       vx

                                          vx
               vy                                                             vx



                           vx




                                                  v = the size of the velocity of the projectile
  To determine the velocity:    vx                vx = the horizontal component of the
                                     vy                projectile’s velocity
  add the horizontal and
                                v                 vy = the vertical component of the
  vertical components
                                                       projectile's velocity
Monday, 24 May 2010
PROJECTILES - HORIZONTAL AND VERTICAL MOTION
• A projectile is an object that has constant horizontal velocity and constant vertical
  acceleration due to gravity. In other words it is moving horizontally at the same time it is in
  free fall. The projectiles velocity is the sum of these two velocities.
• The path of a projectile is parabolic and symmetrical
• Air resistance is considered to be negligible
• The force of gravity is the only force acting on the object. Once the projectile is launched there
  is no thrust force.

                                vy
                                                       vx

                                          vx
               vy                                                             vx



                           vx




                                                  v = the size of the velocity of the projectile
  To determine the velocity:    vx                vx = the horizontal component of the
                                     vy                projectile’s velocity
  add the horizontal and
                                v                 vy = the vertical component of the
  vertical components
                                                       projectile's velocity
Monday, 24 May 2010
PROJECTILES - HORIZONTAL AND VERTICAL MOTION
• A projectile is an object that has constant horizontal velocity and constant vertical
  acceleration due to gravity. In other words it is moving horizontally at the same time it is in
  free fall. The projectiles velocity is the sum of these two velocities.
• The path of a projectile is parabolic and symmetrical
• Air resistance is considered to be negligible
• The force of gravity is the only force acting on the object. Once the projectile is launched there
  is no thrust force.

                                vy
                                                        vx

                                          vx   vy = 0
               vy                                                              vx



                           vx




                                                   v = the size of the velocity of the projectile
  To determine the velocity:    vx                 vx = the horizontal component of the
                                     vy                 projectile’s velocity
  add the horizontal and
                                v                  vy = the vertical component of the
  vertical components
                                                        projectile's velocity
Monday, 24 May 2010
PROJECTILES - HORIZONTAL AND VERTICAL MOTION
• A projectile is an object that has constant horizontal velocity and constant vertical
  acceleration due to gravity. In other words it is moving horizontally at the same time it is in
  free fall. The projectiles velocity is the sum of these two velocities.
• The path of a projectile is parabolic and symmetrical
• Air resistance is considered to be negligible
• The force of gravity is the only force acting on the object. Once the projectile is launched there
  is no thrust force.

                                vy
                                                        vx

                                          vx   vy = 0
               vy                                                              vx

                                                                   vy
                           vx




                                                   v = the size of the velocity of the projectile
  To determine the velocity:    vx                 vx = the horizontal component of the
                                     vy                 projectile’s velocity
  add the horizontal and
                                v                  vy = the vertical component of the
  vertical components
                                                        projectile's velocity
Monday, 24 May 2010
PROJECTILES - HORIZONTAL AND VERTICAL MOTION
• A projectile is an object that has constant horizontal velocity and constant vertical
  acceleration due to gravity. In other words it is moving horizontally at the same time it is in
  free fall. The projectiles velocity is the sum of these two velocities.
• The path of a projectile is parabolic and symmetrical
• Air resistance is considered to be negligible
• The force of gravity is the only force acting on the object. Once the projectile is launched there
  is no thrust force.

                                 vy
                                                         vx

                                           vx   vy = 0
               vy       v                                                       vx

                                                                    vy
                            vx




                                                    v = the size of the velocity of the projectile
  To determine the velocity:     vx                 vx = the horizontal component of the
                                      vy                 projectile’s velocity
  add the horizontal and
                                 v                  vy = the vertical component of the
  vertical components
                                                         projectile's velocity
Monday, 24 May 2010
PROJECTILES - HORIZONTAL AND VERTICAL MOTION
• A projectile is an object that has constant horizontal velocity and constant vertical
  acceleration due to gravity. In other words it is moving horizontally at the same time it is in
  free fall. The projectiles velocity is the sum of these two velocities.
• The path of a projectile is parabolic and symmetrical
• Air resistance is considered to be negligible
• The force of gravity is the only force acting on the object. Once the projectile is launched there
  is no thrust force.

                                 vy        v
                                                         vx

                                           vx   vy = 0
               vy       v                                                       vx

                                                                    vy
                            vx




                                                    v = the size of the velocity of the projectile
  To determine the velocity:     vx                 vx = the horizontal component of the
                                      vy                 projectile’s velocity
  add the horizontal and
                                 v                  vy = the vertical component of the
  vertical components
                                                         projectile's velocity
Monday, 24 May 2010
PROJECTILES - HORIZONTAL AND VERTICAL MOTION
• A projectile is an object that has constant horizontal velocity and constant vertical
  acceleration due to gravity. In other words it is moving horizontally at the same time it is in
  free fall. The projectiles velocity is the sum of these two velocities.
• The path of a projectile is parabolic and symmetrical
• Air resistance is considered to be negligible
• The force of gravity is the only force acting on the object. Once the projectile is launched there
  is no thrust force.

                                 vy        v             v
                                                             vx

                                           vx   vy = 0
               vy       v                                                       vx

                                                                    vy
                            vx




                                                    v = the size of the velocity of the projectile
  To determine the velocity:     vx                 vx = the horizontal component of the
                                      vy                 projectile’s velocity
  add the horizontal and
                                 v                  vy = the vertical component of the
  vertical components
                                                         projectile's velocity
Monday, 24 May 2010
PROJECTILES - HORIZONTAL AND VERTICAL MOTION
• A projectile is an object that has constant horizontal velocity and constant vertical
  acceleration due to gravity. In other words it is moving horizontally at the same time it is in
  free fall. The projectiles velocity is the sum of these two velocities.
• The path of a projectile is parabolic and symmetrical
• Air resistance is considered to be negligible
• The force of gravity is the only force acting on the object. Once the projectile is launched there
  is no thrust force.

                                 vy        v             v
                                                             vx

                                           vx   vy = 0
               vy       v                                                       vx

                                                                    vy          v
                            vx




                                                    v = the size of the velocity of the projectile
  To determine the velocity:     vx                 vx = the horizontal component of the
                                      vy                 projectile’s velocity
  add the horizontal and
                                 v                  vy = the vertical component of the
  vertical components
                                                         projectile's velocity
Monday, 24 May 2010
SYMMETRICAL FLIGHT PATH

• The path of a projectile is parabolic and symmetrical

   Symmetrical Path
   It is often convenient to work with half the flight path of a projectile.




   Remember
   When the projectile is at the maximum height the vertical velocity is zero.

                             At t :     vy = 0
                                 2

Monday, 24 May 2010
SYMMETRICAL FLIGHT PATH

• The path of a projectile is parabolic and symmetrical

   Symmetrical Path
   It is often convenient to work with half the flight path of a projectile.




                                       d




   Remember
   When the projectile is at the maximum height the vertical velocity is zero.

                             At t :     vy = 0
                                 2

Monday, 24 May 2010
SYMMETRICAL FLIGHT PATH

• The path of a projectile is parabolic and symmetrical

   Symmetrical Path
   It is often convenient to work with half the flight path of a projectile.




                                      t
                                      2




                                       d




   Remember
   When the projectile is at the maximum height the vertical velocity is zero.

                             At t :       vy = 0
                                 2

Monday, 24 May 2010
SYMMETRICAL FLIGHT PATH

• The path of a projectile is parabolic and symmetrical

   Symmetrical Path
   It is often convenient to work with half the flight path of a projectile.




                                      t
                                      2


                                                                    t

                                       d




   Remember
   When the projectile is at the maximum height the vertical velocity is zero.

                             At t :       vy = 0
                                 2

Monday, 24 May 2010
SYMMETRICAL FLIGHT PATH

• The path of a projectile is parabolic and symmetrical

   Symmetrical Path
   It is often convenient to work with half the flight path of a projectile.




                                      t
                                      2


                                                                    t

                                       d


                                      d=    the total distance travelled by a projectile
                                      t =   the time taken to travel that distance
                                      t/2 = the time taken to reach maximum height

   Remember
   When the projectile is at the maximum height the vertical velocity is zero.

                             At t :       vy = 0
                                 2

Monday, 24 May 2010
Forces                                    The force due to gravity is the only force
                                              acting on the projectile.


                                                 Not a projectile:
                                                 An object in horizontal flight is not a projectile
                  Fgrav                          because Fgrav is balanced by the lift force. An
                                                 object that has not been launched but maintains
                                                 a high horizontal speed will have air friction
                                                 acting on it and therefore must have a thrust
                                                 force to balance air friction. This will not be a
                                                 projectile.

   Vertical motion calculations
• Since the force of gravity is constant (10 N per kg of any object), the acceleration is
  also constant. (10 ms-2 for any object)

• Kinematic equations must be used for analysing the vertical motion


   Horizontal motion calculations
• A projectile progresses horizontally with constant speed.
• Horizontal speed can be calculated in the usual way:



                 Speed = distance travelled   v=d
                         time taken             t           Units of v: ms-1
Monday, 24 May 2010
PROCESS FOR PROBLEM-SOLVING                          [PIA]


1. Read the question and underline the relevant information
2. Draw a diagram of the situation
3. List the information that relates to the vertical motion and the horizontal motion.
   (Keeping it separate. Use a table with two columns “V” & “H”, remembering that
   vertical and horizontal motion are independent of each other)
4. Show up as a positive direction and down as negative. The signs that you use for
   the vertical information should reflect this.
5. Select the appropriate kinematic equation when calculating a vertical quantity.
6. Use v = d/t for calculation of a horizontal quantity.

Example 1
Imagine a car is driven off a 250m cliff at 30 ms-1. How far from the base of the cliff
will the car be when it lands?




                      Diagram

Monday, 24 May 2010
Example 2
A hockey ball is flicked with an initial velocity of 14 ms-1, 45o to the horizontal, as
shown. A spectator 1.6 m tall is standing directly in the path of the ball, 20 m away.
Will the ball hit the spectator?




                                                                                          Diagram




Monday, 24 May 2010
12 PHYSICS            PROJECTILES ASSIGNMENT       Name ______________________
1. A rock is dropped from the top of a bridge. It takes 4 seconds to reach the water
   below.
   (b) Explain why this is not an example of projectile motion.
       ________________________________________________________________
       ________________________________________________________________
   (c) What would need to be done to the rock to turn it into a projectile?
       ________________________________________________________________
   (d) How far has the rock fallen?
       ________________________________________________________________
       ________________________________________________________________
       ________________________________________________________________
   (e) A second rock is now thrown horizontally at 6 ms-1 from the same bridge.
       ________________________________________________________________
   (f) How long will it take for the rock to reach the water?
       ________________________________________________________________
       ________________________________________________________________
       ________________________________________________________________
   (g) Calculate the range of this second rock
       ________________________________________________________________
       ________________________________________________________________
       ________________________________________________________________
Monday, 24 May 2010
2. A 2 kg bowling ball is launched from ground level and follows the path shown
   below:




  At each of the labelled positions A, B, C and D, state the size and direction of:
  (a) the acceleration of the projectile
  (b) the net force on the projectile

3. When an aircraft was travelling in level flight at 200 ms-1, a nut fell off part of the
   landing gear. Assume air friction is negligible.
   (a) Sketch the paths of the aircraft and the nut.




                                                                                        Sketch


  The nut covered a total horizontal distance of 3.0 km.
  (b) Find the total time it took to fall to the ground.
      ________________________________________________________________
      ________________________________________________________________
Monday, 24 May 2010
(c) What was the altitude of the plane when the nut fell off?
       ________________________________________________________________
       ________________________________________________________________
       ________________________________________________________________
       ________________________________________________________________

4. A ball is kicked with an initial velocity as shown below. The angle of inclination is
   40o.


                               21 ms-1


                                  40o


  (a) Calculate the horizontal component of the initial velocity. __________________
  (b) Calculate the vertical component of the initial velocity. ____________________
  (c) What is the instantaneous velocity of the ball at the top of its path?
      ________________________________________________________________
      ________________________________________________________________
      ________________________________________________________________
  (d) Find the height of the ball at the top of its path.
      ________________________________________________________________
      ________________________________________________________________
      ________________________________________________________________
Monday, 24 May 2010
(e) Find the time taken for the ball to reach its maximum vertical displacement.
      ________________________________________________________________
      ________________________________________________________________
      ________________________________________________________________
   (f) Calculate the range of the ball.
       ________________________________________________________________
       ________________________________________________________________
       If the ball was kicked at an angle of 60o to the horizontal, would it have travelled
       as far along the field?
       ________________________________________________________________
       ________________________________________________________________
       ________________________________________________________________
5. A potato was launched from the muzzle of a spud gun with an initial velocity of
   30 ms-1 at an angle of 60o to the horizontal. Will it clear a 25 m tree that is 60 m
   away on the flight path?
   ___________________________________________________________________
   ___________________________________________________________________
   ___________________________________________________________________
6. A cricket ball was hit with the following trajectory. Find the initial velocity of the ball.


                                    25 m



                                            100 m
Monday, 24 May 2010
7. Johnny is competing in the javelin event of his school athletics competition. The
   javelin behaves like an ideal projectile.




   (a) Describe the shape of the path of the javelin.
       ________________________________________________________________
   (b) Ignoring air resistance, draw arrow(s) on the drawing of the javelin below to
       show the force(s) acting on it when it is in the position shown. Name the
       forces.



          Joe now throws the javelin into the air at an angle of 40o above the horizontal at
          an initial velocity of 30 ms-1.




             Joe now throws the javelin into the air at an angle of 40° above the horizontal at an initial velocity of 30 m s–1

Monday, 24 May 2010
(c) Show that the horizontal component of the initial velocity of the javelin is
       23 ms-1.
       ________________________________________________________________
       ________________________________________________________________
   (d) Calculate the range of the javelin under these conditions.
       ________________________________________________________________
       ________________________________________________________________
       ________________________________________________________________
       ________________________________________________________________
       ________________________________________________________________
        [this is a 2004 NCEA exam question]




Monday, 24 May 2010
UNIFORM CIRCULAR MOTION
1. Recognize that even though a body in circular motion may have a constant speed,
   its velocity is changing and hence it is accelerating.

2. Demonstrate that the acceleration is towards the centre, i.e. centripetal.
3. Deduce that all things in circular motion must have a centripetal force.
4. Show graphically using velocity vectors that the acceleration is towards the centre.
5. Remember the equations for uniform circular motion questions, and use them to
   solve problems:




                                                               Read
                                                               Chapter 12 (p139 to 145)
Monday, 24 May 2010
BRAINSTORM




                                      Give me some
                                  everyday examples of
                                    objects in uniform
                                     circular motion




Complete the functional definition below:



An object in uniform circular object is _____________________________________
___________________________________________________________________


Monday, 24 May 2010
CENTRIPETAL ACCELERATION


For an object in uniform circular motion:

• The speed is always the same
• but the direction is always changing


  Therefore the velocity is always changing and so the object is always
  accelerating

                                                              “remember that
                                                              acceleration is the rate
                                                              of change in velocity.”




      REMEMBER
                            Centripetal acceleration:
      THIS -->
                             • Constant in size
                             • Direction is always changing
                             • Direction is towards the centre of the circular
                               path
Monday, 24 May 2010
MORE EVIDENCE FOR AN ACCELERATION TOWARDS THE CENTRE
         For each 0.1 s time interval, determine ∆v. When you carry out the vector
                                                  ~
                     ~
         subtractions required, do so by leaving vf in the position drawn (below).
                                                 ~
       ~
       ~


                                                                        ∆t = 0.01 s

                                                                         vi
                                                                         ~
                      vf
                      ~
                                              .


                vi
                ~
                                                                         vf
                                                                         ~


   ∆t = 0.01 s




Comment on the size and direction of the change in velocity vector:
Monday, 24 May 2010
CENTRIPETAL FORCE


         “Acceleration is caused by an unbalanced force so now we can say something
         about the force acting on an object in circular motion”




                                  Centripetal force:

                                  • Constant in size
           REMEMBER
                                  • Direction is always changing
           THIS -->
                                  • Direction is towards the centre of the circular
                                     path



           “Does this sound familiar??”




    Note
•   Greater speeds will require greater forces.
•   Force in uniform circular motion changes direction not speed
Monday, 24 May 2010
The carousel              WHAT DO WE MEAN BY UNBALANCED FORCE?




 The force acting
 through the rope

  Tension




 The force acting
 vertically downwards

  Gravity



         “Tension on its own does not cause circular motion”          UNBALANCED
                                                                      FORCE
         “Gravity on its own does not cause circular motion”



                      • A combination of these two forces is called the
                        unbalanced force.
                      • It is the unbalanced force that causes the uniform
                        circular motion
Monday, 24 May 2010
EQUATIONS FOR UNIFORM CIRCULAR

Consider a mass in uniform circular motion with speed v and a radius of circular path, r:

                                      v = velocity (ms-1)
                               v      ~
                               ~
     m
                                      F = Force (N)
                      F               ~
                      ~
                               P      a = acceleration (ms-2)
                          a           ~
                          ~
                 r                    r   = radius of the circular path (m)
                                            (measured from the centre of the path to
                                            the centre of mass of the object.
                                      m = mass of the object (kg)


                                              velocity is at a tangent to
                              At any point:
                                              the circular path


  Speed
  The speed is given by the distance travelled around the circular path divided by
  the time taken to travel that distance.

                                              v=d
                                                 t
Monday, 24 May 2010
Note that the distance travelled in a complete rotation is the circumference, C.
C = 2 πr


   Period and frequency
• The Period, T is the time the object takes to move through one complete rotation.
   (Unit: second, s)


• The frequency, f is the number of revolutions performed per second.
   (Unit: Hertz, Hz or s-1))

                                            T=1                      f=1
                                                         and
                                              f                        T


 Acceleration and force

 Experiment shows that:
                               ac = v2                    ac = centripetal acceleration
                                    r
                                                          Fc = centripetal force
                               Fc = mv2
                                    r     since F = ma

Monday, 24 May 2010
Examples
1. The Apollo 11 space capsule was placed in a parking orbit around the Earth before
   moving onwards to the Moon. The radius of the orbit was 6.56 x 104 m and the
   mass of the capsule was 4.4 x 104 kg.

                                                 to the moon
                                                                                 moon




                      parking orbit   Earth




   (a) If the centripetal force on the capsule was 407 kN while it was in the parking
       orbit, what was its acceleration?
   (b) What was the speed of the capsule in the parking orbit?
   (c) How long did it take the capsule to complete one orbit?




Monday, 24 May 2010
2. A game of swing-ball is played with a 100 g ball. The effective radius of the circular
   path of the ball is 1.4 m. Find the tension in the string (centripetal force) when the
   ball has a velocity of:
   (a) 7.5 ms-1

   (b) 15 ms-1

3. A string has a breaking strain of 320 N. Find the maximum speed that a mass can
   be whirled around in uniform circular motion with a radius of 0.45 m if the mass is
   0.2 kg.




4. An object is in uniform circular motion, tracing an angle of 30o every 0.010 s. Find:
  (a) the period of this motion.

  (b) the frequency of this motion

  If the radius of the object’s path is doubled but the period remains the same, what
  happens to:
  (c) its speed?


   (d) its acceleration?



Monday, 24 May 2010
5. In a circular motion experiment, a mass is whirled around a horizontal circle of
   radius 0.5 m. A student times four revolutions to take 1.5 s.



                                                                              sinker
                                                      tube



  (a) Calculate the speed of the mass around the circle


  (b) What is the direction of the velocity of the mass?


  (c) Calculate the centripetal acceleration of the mass.



  (d) What is the direction of this acceleration?



  (e) How does the value of the centripetal acceleration compare to the acceleration of
      gravity?


                                                                        Ex.12A All Q’s
Monday, 24 May 2010
Monday, 24 May 2010
Monday, 24 May 2010
Monday, 24 May 2010
Monday, 24 May 2010
Monday, 24 May 2010
Monday, 24 May 2010
Monday, 24 May 2010
Monday, 24 May 2010
ANSWERS




Monday, 24 May 2010
12 PHYSICS            CIRCULAR MOTION ASSIGNMENT              Name

1. A car is travelling around a bend in the road and for a few seconds is in uniform
   circular motion.
   (a) The centripetal force is being provided by the road. Name this force.
       ________________________________________________________________
   The car passes over a patch of oil while it is rounding the bend.
   (b) Describe the path the car will take after it hits the oil patch and explain why this
       happens in terms of the forces acting.
       ________________________________________________________________
       ________________________________________________________________
       ________________________________________________________________


2. An object in uniform circular motion completes 10 revolutions in 0.4 seconds
   (a) Find the frequency of this motion.
       ________________________________________________________________
       ________________________________________________________________
   (b) Find the period of this motion.
       ________________________________________________________________
        _______________________________________________________________



Monday, 24 May 2010
3. A big wheel at a fair spins in a circular path of radius 20 m. Once the wheel has
   reached a steady speed, a student times each revolution at 13 seconds.
  (a) Calculate the circumference of the big wheel.
      ______________________________________
  (b) Hence calculate the speed of the big wheel.
      ______________________________________
        ______________________________________
  (c) Calculate the centripetal acceleration of each passenger.
      ________________________________________________________________
      ________________________________________________________________

4. In a circular motion experiment, a mass is whirled around a horizontal circle which
   has a 0.50 m radius. A student time 4 revolutions to take 2.0 s.

   (a) Calculate the speed of the mass around the circle.
       ________________________________________
       ________________________________________
   (b) What is the direction of the velocity of the mass?
       ________________________________________
       ________________________________________
   (c) Calculate the centripetal acceleration of the mass.
       ________________________________________
       ________________________________________

Monday, 24 May 2010
(d) What is the direction of this acceleration?
        _______________________________________________________________
    (e) How does the value of the centripetal acceleration compare to the acceleration
        of gravity? ______________________________________________________

5. Two students go to a fun park for a day where they pay to drive carts around a
   circular track. The track has a radius of 31.8 m and once the carts are at a
   maximum speed they complete a lap in 16 s.

     (a) What is the frequency of the cart’s motion when
         travelling at maximum speed?
         __________________________________________
     (b) When travelling at maximum speed, calculate the
         speed of the cart.
         __________________________________________
         __________________________________________

     (c) Calculate the acceleration of the cart when travelling at maximum speed.
         _____________________________________________________________
         _____________________________________________________________
         _____________________________________________________________
         _____________________________________________________________

Monday, 24 May 2010
The cart has a mass of 150 kg and one of the students, Chris has a mass of 75 kg.
  (d) Calculate the size of the force acting on Chris and his cart at maximum speed.
      ________________________________________________________________
      ________________________________________________________________
      ________________________________________________________________
  (e) Chris drives over a patch of oil and loses control of his cart whilst travelling at
      this maximum speed. On the diagram, draw his path after driving through the
      oil.

6. Jon and Ana are two ice-skaters. In a practiced skating move, Jon spins Ana around
   in a horizontal circle.
                                                  Ana moves in a circle
                                                  as shown:                      Jon

                                                                                   Ana


   (a) Draw an arrow on the diagram to show the direction of the tension force that
       Jon’s arm exerts on Ana at the instant shown.
   (b) If the radius of the circle is 0.95 m and the tension force in Jon’s arm is
       5.00 x 102 N, calculate the speed with which Ana (55 kg) is travelling around the
       circle. Give your answer to the correct number of significant figures.
       ________________________________________________________________
       ________________________________________________________________
       ________________________________________________________________
Monday, 24 May 2010
(d) While Ana is still moving in a circle on the ice, Jon lets her go.
   (i) Describe her velocity (speed and direction) after he releases her.
       ______________________________________________________________
   (ii) Explain why Ana travels with this velocity.
        ______________________________________________________________
        ______________________________________________________________
        ______________________________________________________________
        ______________________________________________________________




Monday, 24 May 2010

More Related Content

What's hot

ملحق اللغة الإنجليزية 2ع ت1
ملحق اللغة الإنجليزية 2ع ت1ملحق اللغة الإنجليزية 2ع ت1
ملحق اللغة الإنجليزية 2ع ت1أمنية وجدى
 
Class 9 physics chapter 1 physics
Class 9 physics chapter 1 physicsClass 9 physics chapter 1 physics
Class 9 physics chapter 1 physicsVinay Kumar Maurya
 
Prospective and retrospective in semantics
Prospective and retrospective in semanticsProspective and retrospective in semantics
Prospective and retrospective in semanticsdarususanti
 
Introduction to inductive and deductive reasoning
Introduction to inductive and deductive reasoningIntroduction to inductive and deductive reasoning
Introduction to inductive and deductive reasoningrbangerter
 
Displacement and Velocity
Displacement and VelocityDisplacement and Velocity
Displacement and Velocitymlong24
 
Physics 2 LT3: Projectile Motion Solutions
Physics 2 LT3: Projectile Motion SolutionsPhysics 2 LT3: Projectile Motion Solutions
Physics 2 LT3: Projectile Motion SolutionsDarwin Quinsaat
 
Newton’s laws of motion
Newton’s laws of motionNewton’s laws of motion
Newton’s laws of motionJimnaira Abanto
 
All About Prepositions.Pdf
All About Prepositions.PdfAll About Prepositions.Pdf
All About Prepositions.PdfJoshua Gorinson
 
Projectile motion 2
Projectile motion 2Projectile motion 2
Projectile motion 2mattvijay
 
Reflexive verbs and pronouns
Reflexive verbs and pronounsReflexive verbs and pronouns
Reflexive verbs and pronounsktlorenzo
 
01 azar grammar present simple vs present continious
01 azar grammar present simple vs present continious01 azar grammar present simple vs present continious
01 azar grammar present simple vs present continiousLilia Rudnitskaya
 

What's hot (20)

Adverb
AdverbAdverb
Adverb
 
ملحق اللغة الإنجليزية 2ع ت1
ملحق اللغة الإنجليزية 2ع ت1ملحق اللغة الإنجليزية 2ع ت1
ملحق اللغة الإنجليزية 2ع ت1
 
speed and velocity
speed and velocityspeed and velocity
speed and velocity
 
Preposition
PrepositionPreposition
Preposition
 
Adverb Clause.pptx
Adverb Clause.pptxAdverb Clause.pptx
Adverb Clause.pptx
 
Class 9 physics chapter 1 physics
Class 9 physics chapter 1 physicsClass 9 physics chapter 1 physics
Class 9 physics chapter 1 physics
 
Logical fallacies
Logical fallaciesLogical fallacies
Logical fallacies
 
Prospective and retrospective in semantics
Prospective and retrospective in semanticsProspective and retrospective in semantics
Prospective and retrospective in semantics
 
direct and indirect.pptx
direct and indirect.pptxdirect and indirect.pptx
direct and indirect.pptx
 
Adverb manner slide
Adverb manner slideAdverb manner slide
Adverb manner slide
 
Introduction to inductive and deductive reasoning
Introduction to inductive and deductive reasoningIntroduction to inductive and deductive reasoning
Introduction to inductive and deductive reasoning
 
Displacement and Velocity
Displacement and VelocityDisplacement and Velocity
Displacement and Velocity
 
Physics 2 LT3: Projectile Motion Solutions
Physics 2 LT3: Projectile Motion SolutionsPhysics 2 LT3: Projectile Motion Solutions
Physics 2 LT3: Projectile Motion Solutions
 
Newton’s laws of motion
Newton’s laws of motionNewton’s laws of motion
Newton’s laws of motion
 
All About Prepositions.Pdf
All About Prepositions.PdfAll About Prepositions.Pdf
All About Prepositions.Pdf
 
Projectile motion 2
Projectile motion 2Projectile motion 2
Projectile motion 2
 
Reflexive verbs and pronouns
Reflexive verbs and pronounsReflexive verbs and pronouns
Reflexive verbs and pronouns
 
Prefixes
PrefixesPrefixes
Prefixes
 
3 - Projectiles
3 - Projectiles3 - Projectiles
3 - Projectiles
 
01 azar grammar present simple vs present continious
01 azar grammar present simple vs present continious01 azar grammar present simple vs present continious
01 azar grammar present simple vs present continious
 

Viewers also liked

2 - Accelerated motion
2 - Accelerated motion2 - Accelerated motion
2 - Accelerated motionguest924dbe
 
Tomografia Computadorizada - Introdução
Tomografia Computadorizada - IntroduçãoTomografia Computadorizada - Introdução
Tomografia Computadorizada - Introduçãocelais0814
 
Proactive Contact Beta Results & Outbound Contact Express
Proactive Contact Beta Results & Outbound Contact ExpressProactive Contact Beta Results & Outbound Contact Express
Proactive Contact Beta Results & Outbound Contact ExpressDavid Ward
 
Overview of Asset Management Firms
Overview of Asset Management Firms Overview of Asset Management Firms
Overview of Asset Management Firms Floyd Saunders
 
Value Engineering And Value Analysis
Value Engineering And Value AnalysisValue Engineering And Value Analysis
Value Engineering And Value Analysisthombremahesh
 
Escritorios remotos
Escritorios remotosEscritorios remotos
Escritorios remotosmestresemhd
 
Iluminacao publica trifasica refinaria
Iluminacao publica trifasica refinariaIluminacao publica trifasica refinaria
Iluminacao publica trifasica refinariaptharsocastro
 
Servo Sistemas Dimensionamento
Servo Sistemas DimensionamentoServo Sistemas Dimensionamento
Servo Sistemas DimensionamentoAlaor Saccomano
 
NIC.br e os novos gTLDs
NIC.br e os novos gTLDsNIC.br e os novos gTLDs
NIC.br e os novos gTLDsRubens Kuhl
 
Modelos hidráulicos a escala
Modelos hidráulicos a escalaModelos hidráulicos a escala
Modelos hidráulicos a escalaAguasSalle
 
Modelamento e Simulação de Redes de Telecomunicações
Modelamento e Simulação de Redes de TelecomunicaçõesModelamento e Simulação de Redes de Telecomunicações
Modelamento e Simulação de Redes de TelecomunicaçõesAntonio Marcos Alberti
 
Zero Paper - Apresentação
Zero Paper - ApresentaçãoZero Paper - Apresentação
Zero Paper - ApresentaçãoPriscila Ota
 
Decisões de investimento
Decisões de investimentoDecisões de investimento
Decisões de investimentoDelza
 
Design de Embalagem - Planejamento
Design de Embalagem - PlanejamentoDesign de Embalagem - Planejamento
Design de Embalagem - PlanejamentoOdair Cavichioli
 
iluminacao planta petroquimica projeto
iluminacao planta petroquimica projetoiluminacao planta petroquimica projeto
iluminacao planta petroquimica projetoptharsocastro
 
Infraestrutura de Servidores - Anchieta
Infraestrutura de Servidores - AnchietaInfraestrutura de Servidores - Anchieta
Infraestrutura de Servidores - AnchietaEdilson Feitoza
 
Equipamentos de uma sonda de perfuração
Equipamentos de uma sonda de perfuraçãoEquipamentos de uma sonda de perfuração
Equipamentos de uma sonda de perfuraçãoAnderson Pontes
 

Viewers also liked (20)

2 - Accelerated motion
2 - Accelerated motion2 - Accelerated motion
2 - Accelerated motion
 
Tomografia Computadorizada - Introdução
Tomografia Computadorizada - IntroduçãoTomografia Computadorizada - Introdução
Tomografia Computadorizada - Introdução
 
Proactive Contact Beta Results & Outbound Contact Express
Proactive Contact Beta Results & Outbound Contact ExpressProactive Contact Beta Results & Outbound Contact Express
Proactive Contact Beta Results & Outbound Contact Express
 
Overview of Asset Management Firms
Overview of Asset Management Firms Overview of Asset Management Firms
Overview of Asset Management Firms
 
Value Engineering And Value Analysis
Value Engineering And Value AnalysisValue Engineering And Value Analysis
Value Engineering And Value Analysis
 
POP Catalogue
POP CataloguePOP Catalogue
POP Catalogue
 
Escritorios remotos
Escritorios remotosEscritorios remotos
Escritorios remotos
 
Iluminacao publica trifasica refinaria
Iluminacao publica trifasica refinariaIluminacao publica trifasica refinaria
Iluminacao publica trifasica refinaria
 
Servo Sistemas Dimensionamento
Servo Sistemas DimensionamentoServo Sistemas Dimensionamento
Servo Sistemas Dimensionamento
 
NIC.br e os novos gTLDs
NIC.br e os novos gTLDsNIC.br e os novos gTLDs
NIC.br e os novos gTLDs
 
A Ultima Viagem De Taxi
A Ultima Viagem De TaxiA Ultima Viagem De Taxi
A Ultima Viagem De Taxi
 
Modelos hidráulicos a escala
Modelos hidráulicos a escalaModelos hidráulicos a escala
Modelos hidráulicos a escala
 
Modelamento e Simulação de Redes de Telecomunicações
Modelamento e Simulação de Redes de TelecomunicaçõesModelamento e Simulação de Redes de Telecomunicações
Modelamento e Simulação de Redes de Telecomunicações
 
Zero Paper - Apresentação
Zero Paper - ApresentaçãoZero Paper - Apresentação
Zero Paper - Apresentação
 
Padrões de Arquitetura na Nuvem da AWS
Padrões de Arquitetura na Nuvem da AWSPadrões de Arquitetura na Nuvem da AWS
Padrões de Arquitetura na Nuvem da AWS
 
Decisões de investimento
Decisões de investimentoDecisões de investimento
Decisões de investimento
 
Design de Embalagem - Planejamento
Design de Embalagem - PlanejamentoDesign de Embalagem - Planejamento
Design de Embalagem - Planejamento
 
iluminacao planta petroquimica projeto
iluminacao planta petroquimica projetoiluminacao planta petroquimica projeto
iluminacao planta petroquimica projeto
 
Infraestrutura de Servidores - Anchieta
Infraestrutura de Servidores - AnchietaInfraestrutura de Servidores - Anchieta
Infraestrutura de Servidores - Anchieta
 
Equipamentos de uma sonda de perfuração
Equipamentos de uma sonda de perfuraçãoEquipamentos de uma sonda de perfuração
Equipamentos de uma sonda de perfuração
 

More from matcol

How to look at art 2
How to look at art  2How to look at art  2
How to look at art 2matcol
 
Modernism vs post modernism
Modernism vs post modernismModernism vs post modernism
Modernism vs post modernismmatcol
 
Electromagnetism
ElectromagnetismElectromagnetism
Electromagnetismmatcol
 
Carbon chemistry
Carbon chemistryCarbon chemistry
Carbon chemistrymatcol
 
Our World
Our WorldOur World
Our Worldmatcol
 
Chemical Reactions
Chemical ReactionsChemical Reactions
Chemical Reactionsmatcol
 
Electromagnetism
ElectromagnetismElectromagnetism
Electromagnetismmatcol
 
DC Electricity & Electromagnetism
DC Electricity & ElectromagnetismDC Electricity & Electromagnetism
DC Electricity & Electromagnetismmatcol
 
Static Electricity
Static ElectricityStatic Electricity
Static Electricitymatcol
 
Bond angles question answer
Bond angles question answerBond angles question answer
Bond angles question answermatcol
 
Chemical Reactions
Chemical ReactionsChemical Reactions
Chemical Reactionsmatcol
 
Momentum & Energy
Momentum & EnergyMomentum & Energy
Momentum & Energymatcol
 
Forces & Eqm
Forces & EqmForces & Eqm
Forces & Eqmmatcol
 
Linear Motion
Linear MotionLinear Motion
Linear Motionmatcol
 
Apes Vs Humans & Skeletal Differences
Apes Vs Humans & Skeletal DifferencesApes Vs Humans & Skeletal Differences
Apes Vs Humans & Skeletal Differencesmatcol
 
Geology
GeologyGeology
Geologymatcol
 
11 - DC Electricity
11 - DC Electricity11 - DC Electricity
11 - DC Electricitymatcol
 
Shaky ground
Shaky groundShaky ground
Shaky groundmatcol
 
1 - Revision of Y11 Mechanics
1 - Revision of Y11 Mechanics1 - Revision of Y11 Mechanics
1 - Revision of Y11 Mechanicsmatcol
 

More from matcol (19)

How to look at art 2
How to look at art  2How to look at art  2
How to look at art 2
 
Modernism vs post modernism
Modernism vs post modernismModernism vs post modernism
Modernism vs post modernism
 
Electromagnetism
ElectromagnetismElectromagnetism
Electromagnetism
 
Carbon chemistry
Carbon chemistryCarbon chemistry
Carbon chemistry
 
Our World
Our WorldOur World
Our World
 
Chemical Reactions
Chemical ReactionsChemical Reactions
Chemical Reactions
 
Electromagnetism
ElectromagnetismElectromagnetism
Electromagnetism
 
DC Electricity & Electromagnetism
DC Electricity & ElectromagnetismDC Electricity & Electromagnetism
DC Electricity & Electromagnetism
 
Static Electricity
Static ElectricityStatic Electricity
Static Electricity
 
Bond angles question answer
Bond angles question answerBond angles question answer
Bond angles question answer
 
Chemical Reactions
Chemical ReactionsChemical Reactions
Chemical Reactions
 
Momentum & Energy
Momentum & EnergyMomentum & Energy
Momentum & Energy
 
Forces & Eqm
Forces & EqmForces & Eqm
Forces & Eqm
 
Linear Motion
Linear MotionLinear Motion
Linear Motion
 
Apes Vs Humans & Skeletal Differences
Apes Vs Humans & Skeletal DifferencesApes Vs Humans & Skeletal Differences
Apes Vs Humans & Skeletal Differences
 
Geology
GeologyGeology
Geology
 
11 - DC Electricity
11 - DC Electricity11 - DC Electricity
11 - DC Electricity
 
Shaky ground
Shaky groundShaky ground
Shaky ground
 
1 - Revision of Y11 Mechanics
1 - Revision of Y11 Mechanics1 - Revision of Y11 Mechanics
1 - Revision of Y11 Mechanics
 

Recently uploaded

How to write a Business Continuity Plan
How to write a Business Continuity PlanHow to write a Business Continuity Plan
How to write a Business Continuity PlanDatabarracks
 
DSPy a system for AI to Write Prompts and Do Fine Tuning
DSPy a system for AI to Write Prompts and Do Fine TuningDSPy a system for AI to Write Prompts and Do Fine Tuning
DSPy a system for AI to Write Prompts and Do Fine TuningLars Bell
 
"ML in Production",Oleksandr Bagan
"ML in Production",Oleksandr Bagan"ML in Production",Oleksandr Bagan
"ML in Production",Oleksandr BaganFwdays
 
"Subclassing and Composition – A Pythonic Tour of Trade-Offs", Hynek Schlawack
"Subclassing and Composition – A Pythonic Tour of Trade-Offs", Hynek Schlawack"Subclassing and Composition – A Pythonic Tour of Trade-Offs", Hynek Schlawack
"Subclassing and Composition – A Pythonic Tour of Trade-Offs", Hynek SchlawackFwdays
 
SAP Build Work Zone - Overview L2-L3.pptx
SAP Build Work Zone - Overview L2-L3.pptxSAP Build Work Zone - Overview L2-L3.pptx
SAP Build Work Zone - Overview L2-L3.pptxNavinnSomaal
 
Unraveling Multimodality with Large Language Models.pdf
Unraveling Multimodality with Large Language Models.pdfUnraveling Multimodality with Large Language Models.pdf
Unraveling Multimodality with Large Language Models.pdfAlex Barbosa Coqueiro
 
From Family Reminiscence to Scholarly Archive .
From Family Reminiscence to Scholarly Archive .From Family Reminiscence to Scholarly Archive .
From Family Reminiscence to Scholarly Archive .Alan Dix
 
Scanning the Internet for External Cloud Exposures via SSL Certs
Scanning the Internet for External Cloud Exposures via SSL CertsScanning the Internet for External Cloud Exposures via SSL Certs
Scanning the Internet for External Cloud Exposures via SSL CertsRizwan Syed
 
Dev Dives: Streamline document processing with UiPath Studio Web
Dev Dives: Streamline document processing with UiPath Studio WebDev Dives: Streamline document processing with UiPath Studio Web
Dev Dives: Streamline document processing with UiPath Studio WebUiPathCommunity
 
Transcript: New from BookNet Canada for 2024: BNC CataList - Tech Forum 2024
Transcript: New from BookNet Canada for 2024: BNC CataList - Tech Forum 2024Transcript: New from BookNet Canada for 2024: BNC CataList - Tech Forum 2024
Transcript: New from BookNet Canada for 2024: BNC CataList - Tech Forum 2024BookNet Canada
 
DevoxxFR 2024 Reproducible Builds with Apache Maven
DevoxxFR 2024 Reproducible Builds with Apache MavenDevoxxFR 2024 Reproducible Builds with Apache Maven
DevoxxFR 2024 Reproducible Builds with Apache MavenHervé Boutemy
 
WordPress Websites for Engineers: Elevate Your Brand
WordPress Websites for Engineers: Elevate Your BrandWordPress Websites for Engineers: Elevate Your Brand
WordPress Websites for Engineers: Elevate Your Brandgvaughan
 
"LLMs for Python Engineers: Advanced Data Analysis and Semantic Kernel",Oleks...
"LLMs for Python Engineers: Advanced Data Analysis and Semantic Kernel",Oleks..."LLMs for Python Engineers: Advanced Data Analysis and Semantic Kernel",Oleks...
"LLMs for Python Engineers: Advanced Data Analysis and Semantic Kernel",Oleks...Fwdays
 
Gen AI in Business - Global Trends Report 2024.pdf
Gen AI in Business - Global Trends Report 2024.pdfGen AI in Business - Global Trends Report 2024.pdf
Gen AI in Business - Global Trends Report 2024.pdfAddepto
 
Connect Wave/ connectwave Pitch Deck Presentation
Connect Wave/ connectwave Pitch Deck PresentationConnect Wave/ connectwave Pitch Deck Presentation
Connect Wave/ connectwave Pitch Deck PresentationSlibray Presentation
 
Advanced Test Driven-Development @ php[tek] 2024
Advanced Test Driven-Development @ php[tek] 2024Advanced Test Driven-Development @ php[tek] 2024
Advanced Test Driven-Development @ php[tek] 2024Scott Keck-Warren
 
TeamStation AI System Report LATAM IT Salaries 2024
TeamStation AI System Report LATAM IT Salaries 2024TeamStation AI System Report LATAM IT Salaries 2024
TeamStation AI System Report LATAM IT Salaries 2024Lonnie McRorey
 
"Debugging python applications inside k8s environment", Andrii Soldatenko
"Debugging python applications inside k8s environment", Andrii Soldatenko"Debugging python applications inside k8s environment", Andrii Soldatenko
"Debugging python applications inside k8s environment", Andrii SoldatenkoFwdays
 
SIP trunking in Janus @ Kamailio World 2024
SIP trunking in Janus @ Kamailio World 2024SIP trunking in Janus @ Kamailio World 2024
SIP trunking in Janus @ Kamailio World 2024Lorenzo Miniero
 

Recently uploaded (20)

How to write a Business Continuity Plan
How to write a Business Continuity PlanHow to write a Business Continuity Plan
How to write a Business Continuity Plan
 
DSPy a system for AI to Write Prompts and Do Fine Tuning
DSPy a system for AI to Write Prompts and Do Fine TuningDSPy a system for AI to Write Prompts and Do Fine Tuning
DSPy a system for AI to Write Prompts and Do Fine Tuning
 
"ML in Production",Oleksandr Bagan
"ML in Production",Oleksandr Bagan"ML in Production",Oleksandr Bagan
"ML in Production",Oleksandr Bagan
 
"Subclassing and Composition – A Pythonic Tour of Trade-Offs", Hynek Schlawack
"Subclassing and Composition – A Pythonic Tour of Trade-Offs", Hynek Schlawack"Subclassing and Composition – A Pythonic Tour of Trade-Offs", Hynek Schlawack
"Subclassing and Composition – A Pythonic Tour of Trade-Offs", Hynek Schlawack
 
SAP Build Work Zone - Overview L2-L3.pptx
SAP Build Work Zone - Overview L2-L3.pptxSAP Build Work Zone - Overview L2-L3.pptx
SAP Build Work Zone - Overview L2-L3.pptx
 
Unraveling Multimodality with Large Language Models.pdf
Unraveling Multimodality with Large Language Models.pdfUnraveling Multimodality with Large Language Models.pdf
Unraveling Multimodality with Large Language Models.pdf
 
From Family Reminiscence to Scholarly Archive .
From Family Reminiscence to Scholarly Archive .From Family Reminiscence to Scholarly Archive .
From Family Reminiscence to Scholarly Archive .
 
Scanning the Internet for External Cloud Exposures via SSL Certs
Scanning the Internet for External Cloud Exposures via SSL CertsScanning the Internet for External Cloud Exposures via SSL Certs
Scanning the Internet for External Cloud Exposures via SSL Certs
 
Dev Dives: Streamline document processing with UiPath Studio Web
Dev Dives: Streamline document processing with UiPath Studio WebDev Dives: Streamline document processing with UiPath Studio Web
Dev Dives: Streamline document processing with UiPath Studio Web
 
Transcript: New from BookNet Canada for 2024: BNC CataList - Tech Forum 2024
Transcript: New from BookNet Canada for 2024: BNC CataList - Tech Forum 2024Transcript: New from BookNet Canada for 2024: BNC CataList - Tech Forum 2024
Transcript: New from BookNet Canada for 2024: BNC CataList - Tech Forum 2024
 
DevoxxFR 2024 Reproducible Builds with Apache Maven
DevoxxFR 2024 Reproducible Builds with Apache MavenDevoxxFR 2024 Reproducible Builds with Apache Maven
DevoxxFR 2024 Reproducible Builds with Apache Maven
 
E-Vehicle_Hacking_by_Parul Sharma_null_owasp.pptx
E-Vehicle_Hacking_by_Parul Sharma_null_owasp.pptxE-Vehicle_Hacking_by_Parul Sharma_null_owasp.pptx
E-Vehicle_Hacking_by_Parul Sharma_null_owasp.pptx
 
WordPress Websites for Engineers: Elevate Your Brand
WordPress Websites for Engineers: Elevate Your BrandWordPress Websites for Engineers: Elevate Your Brand
WordPress Websites for Engineers: Elevate Your Brand
 
"LLMs for Python Engineers: Advanced Data Analysis and Semantic Kernel",Oleks...
"LLMs for Python Engineers: Advanced Data Analysis and Semantic Kernel",Oleks..."LLMs for Python Engineers: Advanced Data Analysis and Semantic Kernel",Oleks...
"LLMs for Python Engineers: Advanced Data Analysis and Semantic Kernel",Oleks...
 
Gen AI in Business - Global Trends Report 2024.pdf
Gen AI in Business - Global Trends Report 2024.pdfGen AI in Business - Global Trends Report 2024.pdf
Gen AI in Business - Global Trends Report 2024.pdf
 
Connect Wave/ connectwave Pitch Deck Presentation
Connect Wave/ connectwave Pitch Deck PresentationConnect Wave/ connectwave Pitch Deck Presentation
Connect Wave/ connectwave Pitch Deck Presentation
 
Advanced Test Driven-Development @ php[tek] 2024
Advanced Test Driven-Development @ php[tek] 2024Advanced Test Driven-Development @ php[tek] 2024
Advanced Test Driven-Development @ php[tek] 2024
 
TeamStation AI System Report LATAM IT Salaries 2024
TeamStation AI System Report LATAM IT Salaries 2024TeamStation AI System Report LATAM IT Salaries 2024
TeamStation AI System Report LATAM IT Salaries 2024
 
"Debugging python applications inside k8s environment", Andrii Soldatenko
"Debugging python applications inside k8s environment", Andrii Soldatenko"Debugging python applications inside k8s environment", Andrii Soldatenko
"Debugging python applications inside k8s environment", Andrii Soldatenko
 
SIP trunking in Janus @ Kamailio World 2024
SIP trunking in Janus @ Kamailio World 2024SIP trunking in Janus @ Kamailio World 2024
SIP trunking in Janus @ Kamailio World 2024
 

Non Linear Motion

  • 1. PROJECTILE MOTION constant acceleration in a straight line; free fall under gravity, projectile motion; Relative motion, change in velocity, velocity vector components, circular motion (constant speed with one force only providing centripetal force). 1. Recall that in the absence of friction a falling object will have a constant acceleration of 10 ms-2 and that this value is referred to as “gravitational acceleration, g” 2. Use F = mg to show that Nkg-1 is an equivalent unit to ms-2. 3. Explain the effect that air friction has on the acceleration of a falling object (resulting in free fall) 4. Define the term projectile. 5. Describe projectile motion in terms of its uniform horizontal motion and it accelerated vertical motion. 6. Use equations of motion to calculate time, distance, velocity and acceleration. Monday, 24 May 2010
  • 2. FREE FALL IN THE ABSENCE OF FRICTION Mass & weight The mass of an object, m is a measure of the amount of matter in that object. The weight of an object, Fw is a measure of the force due to gravity on that object. Gravitational constant, g The force exerted on an object by the earth’s gravitational field is called the gravitational constant, g. What g = 10 Nkg -1 does this mean? This means that for every kg of mass there is a force of 10N acting on it because of gravity This causes all objects to fall with an acceleration of 10 ms-2 We can also say g = 10 ms-2 which is called gravitational acceleration Fw = mg Monday, 24 May 2010
  • 3. FOR AN OBJECT THROWN VERTICALLY UPWARDS THE ACCELERATION IS CONSTANT & NEGATIVE It slows down as it moves in the upwards + up (positive) direction and - down speeds up as it moves in the downwards (negative) direction. Velocity - time graph for the object v (ms-1) t (s) Constant negative slope shows a constant negative acceleration Monday, 24 May 2010
  • 4. FOR AN OBJECT THROWN VERTICALLY UPWARDS THE ACCELERATION IS CONSTANT & NEGATIVE It slows down as it moves in the upwards + up (positive) direction and - down speeds up as it moves in the downwards (negative) direction. Velocity - time graph for the object v (ms-1) t (s) Constant negative slope shows a constant negative acceleration Monday, 24 May 2010
  • 5. FOR AN OBJECT THROWN VERTICALLY UPWARDS THE ACCELERATION IS CONSTANT & NEGATIVE It slows down as it moves in the upwards + up (positive) direction and - down speeds up as it moves in the downwards (negative) direction. Velocity - time graph for the object v (ms-1) t (s) Constant negative slope shows a constant negative acceleration Monday, 24 May 2010
  • 6. FOR AN OBJECT THROWN VERTICALLY UPWARDS THE ACCELERATION IS CONSTANT & NEGATIVE It slows down as it moves in the upwards + up (positive) direction and - down speeds up as it moves in the downwards (negative) direction. A symmetrical path Velocity - time graph for the object v (ms-1) t (s) Constant negative slope shows a constant negative acceleration Monday, 24 May 2010
  • 7. FOR AN OBJECT THROWN VERTICALLY UPWARDS THE ACCELERATION IS CONSTANT & NEGATIVE It slows down as it moves in the upwards + up (positive) direction and - down speeds up as it moves in the downwards (negative) direction. A symmetrical path Velocity - time graph for the object v (ms-1) x t (s) Constant negative slope shows a constant negative acceleration Monday, 24 May 2010
  • 8. FOR AN OBJECT THROWN VERTICALLY UPWARDS THE ACCELERATION IS CONSTANT & NEGATIVE It slows down as it moves in the upwards + up (positive) direction and - down speeds up as it moves in the downwards (negative) direction. A symmetrical path Velocity - time graph for the object v (ms-1) x x t (s) Constant negative slope shows a constant negative acceleration Monday, 24 May 2010
  • 9. FOR AN OBJECT THROWN VERTICALLY UPWARDS THE ACCELERATION IS CONSTANT & NEGATIVE It slows down as it moves in the upwards + up (positive) direction and - down speeds up as it moves in the downwards (negative) direction. A symmetrical path Velocity - time graph for the object v (ms-1) x x t (s) Constant negative x slope shows a constant negative acceleration Monday, 24 May 2010
  • 10. FOR AN OBJECT THROWN VERTICALLY UPWARDS THE ACCELERATION IS CONSTANT & NEGATIVE It slows down as it moves in the upwards + up (positive) direction and - down speeds up as it moves in the downwards (negative) direction. A symmetrical path Velocity - time graph for the object v (ms-1) x x t (s) Constant negative x slope shows a constant negative acceleration Monday, 24 May 2010
  • 11. FOR AN OBJECT THROWN VERTICALLY UPWARDS THE ACCELERATION IS CONSTANT & NEGATIVE It slows down as it moves in the upwards + up (positive) direction and - down speeds up as it moves in the downwards (negative) direction. A symmetrical path Velocity - time graph for the object Decreasing speed in v (ms-1) x a positive direction x t (s) Constant negative x slope shows a constant negative acceleration Monday, 24 May 2010
  • 12. FOR AN OBJECT THROWN VERTICALLY UPWARDS THE ACCELERATION IS CONSTANT & NEGATIVE It slows down as it moves in the upwards + up (positive) direction and - down speeds up as it moves in the downwards (negative) direction. A symmetrical path Velocity - time graph for the object Decreasing speed in v (ms-1) x a positive direction Stationary for an instant x t (s) Constant negative x slope shows a constant negative acceleration Monday, 24 May 2010
  • 13. FOR AN OBJECT THROWN VERTICALLY UPWARDS THE ACCELERATION IS CONSTANT & NEGATIVE It slows down as it moves in the upwards + up (positive) direction and - down speeds up as it moves in the downwards (negative) direction. A symmetrical path Velocity - time graph for the object Decreasing speed in v (ms-1) x a positive direction Stationary for an instant x t (s) Constant negative Increasing speed in a slope shows a negative direction x constant negative acceleration Monday, 24 May 2010
  • 14. Examples 1. A plane drops a Red Cross package from a height of 1200 m. If the package had no parachute (and by this you can assume negligible air friction & g = 10 ms-2) (a) How fast will the package be travelling just before it hits the ground? (b) How many seconds will the package take to fall? 2. A 1 kg object is dropped from a tower 120 m high. (a) Calculate the time it will take for the object to fall to the ground. (b) Calculate the objects final speed on reaching the ground. (c) How long does it take to reach a speed of 35 ms-1 ? Monday, 24 May 2010
  • 15. 3. A shell is fired straight up with an initial speed of 96 ms-1. (a) Calculate the time it will take for the object to fall to the ground. (b) When will the shell have an upwards speed of 48 ms-1 ? (c) Calculate the time for the shell to reach its maximum height. (d) Calculate the maximum height reached by the shell. (e) What is the shells acceleration at the top of its motion? Monday, 24 May 2010
  • 16. FREE FALL - IN THE PRESENCE OF FRICTION If the object is falling fast: Air friction is too large to ignore. Here we consider the object to have two forces acting on it. Air friction and gravity. A. When the object is initially falling slowly, air friction is much smaller than the force of gravity on the object. B. As the object speeds up the air friction increases. C. The object will eventually reach a speed at which the air friction balances the force of gravity on the object. The object cannot fall any faster than at this speed (unless it changes shape). This speed is called terminal velocity. This situation is illustrated by the following example: B A C Vector arithmetic with notes Monday, 24 May 2010
  • 17. WHAT IS A PROJECTILE? Brainstorm Examples of Projectiles “What do all these examples have in common?” “Why do you think that a ball dropping vertically (in free fall) is not an example of projectile motion?” Monday, 24 May 2010
  • 18. PROJECTILES - HORIZONTAL AND VERTICAL MOTION • A projectile is an object that has constant horizontal velocity and constant vertical acceleration due to gravity. In other words it is moving horizontally at the same time it is in free fall. The projectiles velocity is the sum of these two velocities. • The path of a projectile is parabolic and symmetrical • Air resistance is considered to be negligible • The force of gravity is the only force acting on the object. Once the projectile is launched there is no thrust force. v = the size of the velocity of the projectile To determine the velocity: vx vx = the horizontal component of the vy projectile’s velocity add the horizontal and v vy = the vertical component of the vertical components projectile's velocity Monday, 24 May 2010
  • 19. PROJECTILES - HORIZONTAL AND VERTICAL MOTION • A projectile is an object that has constant horizontal velocity and constant vertical acceleration due to gravity. In other words it is moving horizontally at the same time it is in free fall. The projectiles velocity is the sum of these two velocities. • The path of a projectile is parabolic and symmetrical • Air resistance is considered to be negligible • The force of gravity is the only force acting on the object. Once the projectile is launched there is no thrust force. v = the size of the velocity of the projectile To determine the velocity: vx vx = the horizontal component of the vy projectile’s velocity add the horizontal and v vy = the vertical component of the vertical components projectile's velocity Monday, 24 May 2010
  • 20. PROJECTILES - HORIZONTAL AND VERTICAL MOTION • A projectile is an object that has constant horizontal velocity and constant vertical acceleration due to gravity. In other words it is moving horizontally at the same time it is in free fall. The projectiles velocity is the sum of these two velocities. • The path of a projectile is parabolic and symmetrical • Air resistance is considered to be negligible • The force of gravity is the only force acting on the object. Once the projectile is launched there is no thrust force. v = the size of the velocity of the projectile To determine the velocity: vx vx = the horizontal component of the vy projectile’s velocity add the horizontal and v vy = the vertical component of the vertical components projectile's velocity Monday, 24 May 2010
  • 21. PROJECTILES - HORIZONTAL AND VERTICAL MOTION • A projectile is an object that has constant horizontal velocity and constant vertical acceleration due to gravity. In other words it is moving horizontally at the same time it is in free fall. The projectiles velocity is the sum of these two velocities. • The path of a projectile is parabolic and symmetrical • Air resistance is considered to be negligible • The force of gravity is the only force acting on the object. Once the projectile is launched there is no thrust force. v = the size of the velocity of the projectile To determine the velocity: vx vx = the horizontal component of the vy projectile’s velocity add the horizontal and v vy = the vertical component of the vertical components projectile's velocity Monday, 24 May 2010
  • 22. PROJECTILES - HORIZONTAL AND VERTICAL MOTION • A projectile is an object that has constant horizontal velocity and constant vertical acceleration due to gravity. In other words it is moving horizontally at the same time it is in free fall. The projectiles velocity is the sum of these two velocities. • The path of a projectile is parabolic and symmetrical • Air resistance is considered to be negligible • The force of gravity is the only force acting on the object. Once the projectile is launched there is no thrust force. v = the size of the velocity of the projectile To determine the velocity: vx vx = the horizontal component of the vy projectile’s velocity add the horizontal and v vy = the vertical component of the vertical components projectile's velocity Monday, 24 May 2010
  • 23. PROJECTILES - HORIZONTAL AND VERTICAL MOTION • A projectile is an object that has constant horizontal velocity and constant vertical acceleration due to gravity. In other words it is moving horizontally at the same time it is in free fall. The projectiles velocity is the sum of these two velocities. • The path of a projectile is parabolic and symmetrical • Air resistance is considered to be negligible • The force of gravity is the only force acting on the object. Once the projectile is launched there is no thrust force. v = the size of the velocity of the projectile To determine the velocity: vx vx = the horizontal component of the vy projectile’s velocity add the horizontal and v vy = the vertical component of the vertical components projectile's velocity Monday, 24 May 2010
  • 24. PROJECTILES - HORIZONTAL AND VERTICAL MOTION • A projectile is an object that has constant horizontal velocity and constant vertical acceleration due to gravity. In other words it is moving horizontally at the same time it is in free fall. The projectiles velocity is the sum of these two velocities. • The path of a projectile is parabolic and symmetrical • Air resistance is considered to be negligible • The force of gravity is the only force acting on the object. Once the projectile is launched there is no thrust force. vx v = the size of the velocity of the projectile To determine the velocity: vx vx = the horizontal component of the vy projectile’s velocity add the horizontal and v vy = the vertical component of the vertical components projectile's velocity Monday, 24 May 2010
  • 25. PROJECTILES - HORIZONTAL AND VERTICAL MOTION • A projectile is an object that has constant horizontal velocity and constant vertical acceleration due to gravity. In other words it is moving horizontally at the same time it is in free fall. The projectiles velocity is the sum of these two velocities. • The path of a projectile is parabolic and symmetrical • Air resistance is considered to be negligible • The force of gravity is the only force acting on the object. Once the projectile is launched there is no thrust force. vx vx v = the size of the velocity of the projectile To determine the velocity: vx vx = the horizontal component of the vy projectile’s velocity add the horizontal and v vy = the vertical component of the vertical components projectile's velocity Monday, 24 May 2010
  • 26. PROJECTILES - HORIZONTAL AND VERTICAL MOTION • A projectile is an object that has constant horizontal velocity and constant vertical acceleration due to gravity. In other words it is moving horizontally at the same time it is in free fall. The projectiles velocity is the sum of these two velocities. • The path of a projectile is parabolic and symmetrical • Air resistance is considered to be negligible • The force of gravity is the only force acting on the object. Once the projectile is launched there is no thrust force. vx vx vx v = the size of the velocity of the projectile To determine the velocity: vx vx = the horizontal component of the vy projectile’s velocity add the horizontal and v vy = the vertical component of the vertical components projectile's velocity Monday, 24 May 2010
  • 27. PROJECTILES - HORIZONTAL AND VERTICAL MOTION • A projectile is an object that has constant horizontal velocity and constant vertical acceleration due to gravity. In other words it is moving horizontally at the same time it is in free fall. The projectiles velocity is the sum of these two velocities. • The path of a projectile is parabolic and symmetrical • Air resistance is considered to be negligible • The force of gravity is the only force acting on the object. Once the projectile is launched there is no thrust force. vx vx vx vx v = the size of the velocity of the projectile To determine the velocity: vx vx = the horizontal component of the vy projectile’s velocity add the horizontal and v vy = the vertical component of the vertical components projectile's velocity Monday, 24 May 2010
  • 28. PROJECTILES - HORIZONTAL AND VERTICAL MOTION • A projectile is an object that has constant horizontal velocity and constant vertical acceleration due to gravity. In other words it is moving horizontally at the same time it is in free fall. The projectiles velocity is the sum of these two velocities. • The path of a projectile is parabolic and symmetrical • Air resistance is considered to be negligible • The force of gravity is the only force acting on the object. Once the projectile is launched there is no thrust force. vx vx vy vx vx v = the size of the velocity of the projectile To determine the velocity: vx vx = the horizontal component of the vy projectile’s velocity add the horizontal and v vy = the vertical component of the vertical components projectile's velocity Monday, 24 May 2010
  • 29. PROJECTILES - HORIZONTAL AND VERTICAL MOTION • A projectile is an object that has constant horizontal velocity and constant vertical acceleration due to gravity. In other words it is moving horizontally at the same time it is in free fall. The projectiles velocity is the sum of these two velocities. • The path of a projectile is parabolic and symmetrical • Air resistance is considered to be negligible • The force of gravity is the only force acting on the object. Once the projectile is launched there is no thrust force. vy vx vx vy vx vx v = the size of the velocity of the projectile To determine the velocity: vx vx = the horizontal component of the vy projectile’s velocity add the horizontal and v vy = the vertical component of the vertical components projectile's velocity Monday, 24 May 2010
  • 30. PROJECTILES - HORIZONTAL AND VERTICAL MOTION • A projectile is an object that has constant horizontal velocity and constant vertical acceleration due to gravity. In other words it is moving horizontally at the same time it is in free fall. The projectiles velocity is the sum of these two velocities. • The path of a projectile is parabolic and symmetrical • Air resistance is considered to be negligible • The force of gravity is the only force acting on the object. Once the projectile is launched there is no thrust force. vy vx vx vy = 0 vy vx vx v = the size of the velocity of the projectile To determine the velocity: vx vx = the horizontal component of the vy projectile’s velocity add the horizontal and v vy = the vertical component of the vertical components projectile's velocity Monday, 24 May 2010
  • 31. PROJECTILES - HORIZONTAL AND VERTICAL MOTION • A projectile is an object that has constant horizontal velocity and constant vertical acceleration due to gravity. In other words it is moving horizontally at the same time it is in free fall. The projectiles velocity is the sum of these two velocities. • The path of a projectile is parabolic and symmetrical • Air resistance is considered to be negligible • The force of gravity is the only force acting on the object. Once the projectile is launched there is no thrust force. vy vx vx vy = 0 vy vx vy vx v = the size of the velocity of the projectile To determine the velocity: vx vx = the horizontal component of the vy projectile’s velocity add the horizontal and v vy = the vertical component of the vertical components projectile's velocity Monday, 24 May 2010
  • 32. PROJECTILES - HORIZONTAL AND VERTICAL MOTION • A projectile is an object that has constant horizontal velocity and constant vertical acceleration due to gravity. In other words it is moving horizontally at the same time it is in free fall. The projectiles velocity is the sum of these two velocities. • The path of a projectile is parabolic and symmetrical • Air resistance is considered to be negligible • The force of gravity is the only force acting on the object. Once the projectile is launched there is no thrust force. vy vx vx vy = 0 vy v vx vy vx v = the size of the velocity of the projectile To determine the velocity: vx vx = the horizontal component of the vy projectile’s velocity add the horizontal and v vy = the vertical component of the vertical components projectile's velocity Monday, 24 May 2010
  • 33. PROJECTILES - HORIZONTAL AND VERTICAL MOTION • A projectile is an object that has constant horizontal velocity and constant vertical acceleration due to gravity. In other words it is moving horizontally at the same time it is in free fall. The projectiles velocity is the sum of these two velocities. • The path of a projectile is parabolic and symmetrical • Air resistance is considered to be negligible • The force of gravity is the only force acting on the object. Once the projectile is launched there is no thrust force. vy v vx vx vy = 0 vy v vx vy vx v = the size of the velocity of the projectile To determine the velocity: vx vx = the horizontal component of the vy projectile’s velocity add the horizontal and v vy = the vertical component of the vertical components projectile's velocity Monday, 24 May 2010
  • 34. PROJECTILES - HORIZONTAL AND VERTICAL MOTION • A projectile is an object that has constant horizontal velocity and constant vertical acceleration due to gravity. In other words it is moving horizontally at the same time it is in free fall. The projectiles velocity is the sum of these two velocities. • The path of a projectile is parabolic and symmetrical • Air resistance is considered to be negligible • The force of gravity is the only force acting on the object. Once the projectile is launched there is no thrust force. vy v v vx vx vy = 0 vy v vx vy vx v = the size of the velocity of the projectile To determine the velocity: vx vx = the horizontal component of the vy projectile’s velocity add the horizontal and v vy = the vertical component of the vertical components projectile's velocity Monday, 24 May 2010
  • 35. PROJECTILES - HORIZONTAL AND VERTICAL MOTION • A projectile is an object that has constant horizontal velocity and constant vertical acceleration due to gravity. In other words it is moving horizontally at the same time it is in free fall. The projectiles velocity is the sum of these two velocities. • The path of a projectile is parabolic and symmetrical • Air resistance is considered to be negligible • The force of gravity is the only force acting on the object. Once the projectile is launched there is no thrust force. vy v v vx vx vy = 0 vy v vx vy v vx v = the size of the velocity of the projectile To determine the velocity: vx vx = the horizontal component of the vy projectile’s velocity add the horizontal and v vy = the vertical component of the vertical components projectile's velocity Monday, 24 May 2010
  • 36. SYMMETRICAL FLIGHT PATH • The path of a projectile is parabolic and symmetrical Symmetrical Path It is often convenient to work with half the flight path of a projectile. Remember When the projectile is at the maximum height the vertical velocity is zero. At t : vy = 0 2 Monday, 24 May 2010
  • 37. SYMMETRICAL FLIGHT PATH • The path of a projectile is parabolic and symmetrical Symmetrical Path It is often convenient to work with half the flight path of a projectile. d Remember When the projectile is at the maximum height the vertical velocity is zero. At t : vy = 0 2 Monday, 24 May 2010
  • 38. SYMMETRICAL FLIGHT PATH • The path of a projectile is parabolic and symmetrical Symmetrical Path It is often convenient to work with half the flight path of a projectile. t 2 d Remember When the projectile is at the maximum height the vertical velocity is zero. At t : vy = 0 2 Monday, 24 May 2010
  • 39. SYMMETRICAL FLIGHT PATH • The path of a projectile is parabolic and symmetrical Symmetrical Path It is often convenient to work with half the flight path of a projectile. t 2 t d Remember When the projectile is at the maximum height the vertical velocity is zero. At t : vy = 0 2 Monday, 24 May 2010
  • 40. SYMMETRICAL FLIGHT PATH • The path of a projectile is parabolic and symmetrical Symmetrical Path It is often convenient to work with half the flight path of a projectile. t 2 t d d= the total distance travelled by a projectile t = the time taken to travel that distance t/2 = the time taken to reach maximum height Remember When the projectile is at the maximum height the vertical velocity is zero. At t : vy = 0 2 Monday, 24 May 2010
  • 41. Forces The force due to gravity is the only force acting on the projectile. Not a projectile: An object in horizontal flight is not a projectile Fgrav because Fgrav is balanced by the lift force. An object that has not been launched but maintains a high horizontal speed will have air friction acting on it and therefore must have a thrust force to balance air friction. This will not be a projectile. Vertical motion calculations • Since the force of gravity is constant (10 N per kg of any object), the acceleration is also constant. (10 ms-2 for any object) • Kinematic equations must be used for analysing the vertical motion Horizontal motion calculations • A projectile progresses horizontally with constant speed. • Horizontal speed can be calculated in the usual way: Speed = distance travelled v=d time taken t Units of v: ms-1 Monday, 24 May 2010
  • 42. PROCESS FOR PROBLEM-SOLVING [PIA] 1. Read the question and underline the relevant information 2. Draw a diagram of the situation 3. List the information that relates to the vertical motion and the horizontal motion. (Keeping it separate. Use a table with two columns “V” & “H”, remembering that vertical and horizontal motion are independent of each other) 4. Show up as a positive direction and down as negative. The signs that you use for the vertical information should reflect this. 5. Select the appropriate kinematic equation when calculating a vertical quantity. 6. Use v = d/t for calculation of a horizontal quantity. Example 1 Imagine a car is driven off a 250m cliff at 30 ms-1. How far from the base of the cliff will the car be when it lands? Diagram Monday, 24 May 2010
  • 43. Example 2 A hockey ball is flicked with an initial velocity of 14 ms-1, 45o to the horizontal, as shown. A spectator 1.6 m tall is standing directly in the path of the ball, 20 m away. Will the ball hit the spectator? Diagram Monday, 24 May 2010
  • 44. 12 PHYSICS PROJECTILES ASSIGNMENT Name ______________________ 1. A rock is dropped from the top of a bridge. It takes 4 seconds to reach the water below. (b) Explain why this is not an example of projectile motion. ________________________________________________________________ ________________________________________________________________ (c) What would need to be done to the rock to turn it into a projectile? ________________________________________________________________ (d) How far has the rock fallen? ________________________________________________________________ ________________________________________________________________ ________________________________________________________________ (e) A second rock is now thrown horizontally at 6 ms-1 from the same bridge. ________________________________________________________________ (f) How long will it take for the rock to reach the water? ________________________________________________________________ ________________________________________________________________ ________________________________________________________________ (g) Calculate the range of this second rock ________________________________________________________________ ________________________________________________________________ ________________________________________________________________ Monday, 24 May 2010
  • 45. 2. A 2 kg bowling ball is launched from ground level and follows the path shown below: At each of the labelled positions A, B, C and D, state the size and direction of: (a) the acceleration of the projectile (b) the net force on the projectile 3. When an aircraft was travelling in level flight at 200 ms-1, a nut fell off part of the landing gear. Assume air friction is negligible. (a) Sketch the paths of the aircraft and the nut. Sketch The nut covered a total horizontal distance of 3.0 km. (b) Find the total time it took to fall to the ground. ________________________________________________________________ ________________________________________________________________ Monday, 24 May 2010
  • 46. (c) What was the altitude of the plane when the nut fell off? ________________________________________________________________ ________________________________________________________________ ________________________________________________________________ ________________________________________________________________ 4. A ball is kicked with an initial velocity as shown below. The angle of inclination is 40o. 21 ms-1 40o (a) Calculate the horizontal component of the initial velocity. __________________ (b) Calculate the vertical component of the initial velocity. ____________________ (c) What is the instantaneous velocity of the ball at the top of its path? ________________________________________________________________ ________________________________________________________________ ________________________________________________________________ (d) Find the height of the ball at the top of its path. ________________________________________________________________ ________________________________________________________________ ________________________________________________________________ Monday, 24 May 2010
  • 47. (e) Find the time taken for the ball to reach its maximum vertical displacement. ________________________________________________________________ ________________________________________________________________ ________________________________________________________________ (f) Calculate the range of the ball. ________________________________________________________________ ________________________________________________________________ If the ball was kicked at an angle of 60o to the horizontal, would it have travelled as far along the field? ________________________________________________________________ ________________________________________________________________ ________________________________________________________________ 5. A potato was launched from the muzzle of a spud gun with an initial velocity of 30 ms-1 at an angle of 60o to the horizontal. Will it clear a 25 m tree that is 60 m away on the flight path? ___________________________________________________________________ ___________________________________________________________________ ___________________________________________________________________ 6. A cricket ball was hit with the following trajectory. Find the initial velocity of the ball. 25 m 100 m Monday, 24 May 2010
  • 48. 7. Johnny is competing in the javelin event of his school athletics competition. The javelin behaves like an ideal projectile. (a) Describe the shape of the path of the javelin. ________________________________________________________________ (b) Ignoring air resistance, draw arrow(s) on the drawing of the javelin below to show the force(s) acting on it when it is in the position shown. Name the forces. Joe now throws the javelin into the air at an angle of 40o above the horizontal at an initial velocity of 30 ms-1. Joe now throws the javelin into the air at an angle of 40° above the horizontal at an initial velocity of 30 m s–1 Monday, 24 May 2010
  • 49. (c) Show that the horizontal component of the initial velocity of the javelin is 23 ms-1. ________________________________________________________________ ________________________________________________________________ (d) Calculate the range of the javelin under these conditions. ________________________________________________________________ ________________________________________________________________ ________________________________________________________________ ________________________________________________________________ ________________________________________________________________ [this is a 2004 NCEA exam question] Monday, 24 May 2010
  • 50. UNIFORM CIRCULAR MOTION 1. Recognize that even though a body in circular motion may have a constant speed, its velocity is changing and hence it is accelerating. 2. Demonstrate that the acceleration is towards the centre, i.e. centripetal. 3. Deduce that all things in circular motion must have a centripetal force. 4. Show graphically using velocity vectors that the acceleration is towards the centre. 5. Remember the equations for uniform circular motion questions, and use them to solve problems: Read Chapter 12 (p139 to 145) Monday, 24 May 2010
  • 51. BRAINSTORM Give me some everyday examples of objects in uniform circular motion Complete the functional definition below: An object in uniform circular object is _____________________________________ ___________________________________________________________________ Monday, 24 May 2010
  • 52. CENTRIPETAL ACCELERATION For an object in uniform circular motion: • The speed is always the same • but the direction is always changing Therefore the velocity is always changing and so the object is always accelerating “remember that acceleration is the rate of change in velocity.” REMEMBER Centripetal acceleration: THIS --> • Constant in size • Direction is always changing • Direction is towards the centre of the circular path Monday, 24 May 2010
  • 53. MORE EVIDENCE FOR AN ACCELERATION TOWARDS THE CENTRE For each 0.1 s time interval, determine ∆v. When you carry out the vector ~ ~ subtractions required, do so by leaving vf in the position drawn (below). ~ ~ ~ ∆t = 0.01 s vi ~ vf ~ . vi ~ vf ~ ∆t = 0.01 s Comment on the size and direction of the change in velocity vector: Monday, 24 May 2010
  • 54. CENTRIPETAL FORCE “Acceleration is caused by an unbalanced force so now we can say something about the force acting on an object in circular motion” Centripetal force: • Constant in size REMEMBER • Direction is always changing THIS --> • Direction is towards the centre of the circular path “Does this sound familiar??” Note • Greater speeds will require greater forces. • Force in uniform circular motion changes direction not speed Monday, 24 May 2010
  • 55. The carousel WHAT DO WE MEAN BY UNBALANCED FORCE? The force acting through the rope Tension The force acting vertically downwards Gravity “Tension on its own does not cause circular motion” UNBALANCED FORCE “Gravity on its own does not cause circular motion” • A combination of these two forces is called the unbalanced force. • It is the unbalanced force that causes the uniform circular motion Monday, 24 May 2010
  • 56. EQUATIONS FOR UNIFORM CIRCULAR Consider a mass in uniform circular motion with speed v and a radius of circular path, r: v = velocity (ms-1) v ~ ~ m F = Force (N) F ~ ~ P a = acceleration (ms-2) a ~ ~ r r = radius of the circular path (m) (measured from the centre of the path to the centre of mass of the object. m = mass of the object (kg) velocity is at a tangent to At any point: the circular path Speed The speed is given by the distance travelled around the circular path divided by the time taken to travel that distance. v=d t Monday, 24 May 2010
  • 57. Note that the distance travelled in a complete rotation is the circumference, C. C = 2 πr Period and frequency • The Period, T is the time the object takes to move through one complete rotation. (Unit: second, s) • The frequency, f is the number of revolutions performed per second. (Unit: Hertz, Hz or s-1)) T=1 f=1 and f T Acceleration and force Experiment shows that: ac = v2 ac = centripetal acceleration r Fc = centripetal force Fc = mv2 r since F = ma Monday, 24 May 2010
  • 58. Examples 1. The Apollo 11 space capsule was placed in a parking orbit around the Earth before moving onwards to the Moon. The radius of the orbit was 6.56 x 104 m and the mass of the capsule was 4.4 x 104 kg. to the moon moon parking orbit Earth (a) If the centripetal force on the capsule was 407 kN while it was in the parking orbit, what was its acceleration? (b) What was the speed of the capsule in the parking orbit? (c) How long did it take the capsule to complete one orbit? Monday, 24 May 2010
  • 59. 2. A game of swing-ball is played with a 100 g ball. The effective radius of the circular path of the ball is 1.4 m. Find the tension in the string (centripetal force) when the ball has a velocity of: (a) 7.5 ms-1 (b) 15 ms-1 3. A string has a breaking strain of 320 N. Find the maximum speed that a mass can be whirled around in uniform circular motion with a radius of 0.45 m if the mass is 0.2 kg. 4. An object is in uniform circular motion, tracing an angle of 30o every 0.010 s. Find: (a) the period of this motion. (b) the frequency of this motion If the radius of the object’s path is doubled but the period remains the same, what happens to: (c) its speed? (d) its acceleration? Monday, 24 May 2010
  • 60. 5. In a circular motion experiment, a mass is whirled around a horizontal circle of radius 0.5 m. A student times four revolutions to take 1.5 s. sinker tube (a) Calculate the speed of the mass around the circle (b) What is the direction of the velocity of the mass? (c) Calculate the centripetal acceleration of the mass. (d) What is the direction of this acceleration? (e) How does the value of the centripetal acceleration compare to the acceleration of gravity? Ex.12A All Q’s Monday, 24 May 2010
  • 70. 12 PHYSICS CIRCULAR MOTION ASSIGNMENT Name 1. A car is travelling around a bend in the road and for a few seconds is in uniform circular motion. (a) The centripetal force is being provided by the road. Name this force. ________________________________________________________________ The car passes over a patch of oil while it is rounding the bend. (b) Describe the path the car will take after it hits the oil patch and explain why this happens in terms of the forces acting. ________________________________________________________________ ________________________________________________________________ ________________________________________________________________ 2. An object in uniform circular motion completes 10 revolutions in 0.4 seconds (a) Find the frequency of this motion. ________________________________________________________________ ________________________________________________________________ (b) Find the period of this motion. ________________________________________________________________ _______________________________________________________________ Monday, 24 May 2010
  • 71. 3. A big wheel at a fair spins in a circular path of radius 20 m. Once the wheel has reached a steady speed, a student times each revolution at 13 seconds. (a) Calculate the circumference of the big wheel. ______________________________________ (b) Hence calculate the speed of the big wheel. ______________________________________ ______________________________________ (c) Calculate the centripetal acceleration of each passenger. ________________________________________________________________ ________________________________________________________________ 4. In a circular motion experiment, a mass is whirled around a horizontal circle which has a 0.50 m radius. A student time 4 revolutions to take 2.0 s. (a) Calculate the speed of the mass around the circle. ________________________________________ ________________________________________ (b) What is the direction of the velocity of the mass? ________________________________________ ________________________________________ (c) Calculate the centripetal acceleration of the mass. ________________________________________ ________________________________________ Monday, 24 May 2010
  • 72. (d) What is the direction of this acceleration? _______________________________________________________________ (e) How does the value of the centripetal acceleration compare to the acceleration of gravity? ______________________________________________________ 5. Two students go to a fun park for a day where they pay to drive carts around a circular track. The track has a radius of 31.8 m and once the carts are at a maximum speed they complete a lap in 16 s. (a) What is the frequency of the cart’s motion when travelling at maximum speed? __________________________________________ (b) When travelling at maximum speed, calculate the speed of the cart. __________________________________________ __________________________________________ (c) Calculate the acceleration of the cart when travelling at maximum speed. _____________________________________________________________ _____________________________________________________________ _____________________________________________________________ _____________________________________________________________ Monday, 24 May 2010
  • 73. The cart has a mass of 150 kg and one of the students, Chris has a mass of 75 kg. (d) Calculate the size of the force acting on Chris and his cart at maximum speed. ________________________________________________________________ ________________________________________________________________ ________________________________________________________________ (e) Chris drives over a patch of oil and loses control of his cart whilst travelling at this maximum speed. On the diagram, draw his path after driving through the oil. 6. Jon and Ana are two ice-skaters. In a practiced skating move, Jon spins Ana around in a horizontal circle. Ana moves in a circle as shown: Jon Ana (a) Draw an arrow on the diagram to show the direction of the tension force that Jon’s arm exerts on Ana at the instant shown. (b) If the radius of the circle is 0.95 m and the tension force in Jon’s arm is 5.00 x 102 N, calculate the speed with which Ana (55 kg) is travelling around the circle. Give your answer to the correct number of significant figures. ________________________________________________________________ ________________________________________________________________ ________________________________________________________________ Monday, 24 May 2010
  • 74. (d) While Ana is still moving in a circle on the ice, Jon lets her go. (i) Describe her velocity (speed and direction) after he releases her. ______________________________________________________________ (ii) Explain why Ana travels with this velocity. ______________________________________________________________ ______________________________________________________________ ______________________________________________________________ ______________________________________________________________ Monday, 24 May 2010